2 Questions About the Gyroscope Effect

  • I
  • Thread starter Johnls
  • Start date
  • Tags
    Gyroscope
In summary, the precession of a gyroscope can be explained by the torque being perpendicular to the angular momentum, which changes in the direction of the torque. The torque can be viewed as a "couple" of equal and opposite forces, and its calculation is independent of the reference point chosen. However, this model does not work for low angular momentums and a more complex three-dimensional treatment is needed. Additionally, the concept of a "nutation" and analysis using tensors is required.
  • #1
Johnls
11
0
I'm talking here about this situation:
phys2_8f_15a.png

The explanation which is usually given as to why there is a precession, is that the torque is perpendicular to the angular momentum and the angular momentum changes in the direction of the torque.
A few things I don't understand about it:

1. The torque is relative to point O but the angular momentum is relative to the center of the wheel. When you derive that torque is the change in the angular momentum, don't you assume that they are both calculated relative to the same point?

2. Why doesn't it work for low angular momentums?
 
Physics news on Phys.org
  • #2
Johnls said:
1. The torque is relative to point O but the angular momentum is relative to the center of the wheel. When you derive that torque is the change in the angular momentum, don't you assume that they are both calculated relative to the same point?
Two points...

A torque can be viewed as a "couple" -- a pair of equal and opposite forces separated by a distance that is not parallel to the forces. It does not matter where you choose to place the reference point when calculating the torque from a couple. You always get the same answer.

Point O lies on the axis of rotation anyway.
2. Why doesn't it work for low angular momentums?

The idea that the angular momentum of a gyroscope is purely horizontal, that it does not deflect downward at all under a vertical force and that the motion associated with the precession has no associated angular momentum is an approximation. For a rapidly spinning gyroscope, it is a good approximation. In first year physics courses one is usually exposed to angular momentum primarily as it applies in two dimensions. In that setting, one considers the gyroscope to be rotating in two dimensions and precessing in the third.

The full three dimensional treatment involves the notion of a "nutation" and analysis using tensors.

One simple way of seeing that the precession model cannot work for low angular momentum is to consider what happens as the rotation rate gets lower and lower. The precession rate gets higher and higher. If you get to a point where the precession rate is higher than the rotation rate, it's pretty clear that you're not considering a gyroscope that is spinning on its intended axis. Instead, it is rotating around a different instantaneous axis. And that axis may keep changing over time.

A gyroscope that is not spinning at all does not precess infinitely fast. It just flops down.
 
  • Like
Likes Johnls
  • #3
jbriggs444 said:
Two points...

A torque can be viewed as a "couple" -- a pair of equal and opposite forces separated by a distance that is not parallel to the forces. It does not matter where you choose to place the reference point when calculating the torque from a couple. You always get the same answer.

Point O lies on the axis of rotation anyway.The idea that the angular momentum of a gyroscope is purely horizontal, that it does not deflect downward at all under a vertical force and that the motion associated with the precession has no associated angular momentum is an approximation. For a rapidly spinning gyroscope, it is a good approximation. In first year physics courses one is usually exposed to angular momentum primarily as it applies in two dimensions. In that setting, one considers the gyroscope to be rotating in two dimensions and precessing in the third.

The full three dimensional treatment involves the notion of a "nutation" and analysis using tensors.

One simple way of seeing that the precession model cannot work for low angular momentum is to consider what happens as the rotation rate gets lower and lower. The precession rate gets higher and higher. If you get to a point where the precession rate is higher than the rotation rate, it's pretty clear that you're not considering a gyroscope that is spinning on its intended axis. Instead, it is rotating around a different instantaneous axis. And that axis may keep changing over time.

A gyroscope that is not spinning at all does not precess infinitely fast. It just flops down.

First of all, thanks for the reply!
Could you please expand a little bit on how torque can be viewed as a "couple"? How can I do this conversion? How does it relate to the Gyro example?
 
  • #4
Johnls said:
First of all, thanks for the reply!
Could you please expand a little bit on how torque can be viewed as a "couple"? How can I do this conversion? How does it relate to the Gyro example?
For the gyroscope, you have an upward force at point O from the support and a downward force from gravity at the center of gravity (a distance r from point O).

Pick any origin you like and compute the torque from the force at point O plus the torque from the force at the center of gravity. Then pick a different origin and compute it again. The two forces are a "couple" and the net torque they produce will be independent of the origin that you pick.
 
  • Like
Likes Johnls
  • #5
jbriggs444 said:
For the gyroscope, you have an upward force at point O from the support and a downward force from gravity at the center of gravity (a distance r from point O).

Pick any origin you like and compute the torque from the force at point O plus the torque from the force at the center of gravity. Then pick a different origin and compute it again. The two forces are a "couple" and the net torque they produce will be independent of the origin that you pick.

Oh I see... Thanks!
 

1. What is the gyroscope effect?

The gyroscope effect, also known as gyroscopic precession, is a phenomenon that occurs when an object with a spinning axis experiences a force applied perpendicular to the axis. This results in the object rotating in a direction perpendicular to both the axis and the applied force.

2. How does the gyroscope effect work?

The gyroscope effect works due to the conservation of angular momentum. When a force is applied perpendicular to the spinning axis of the gyroscope, the resulting torque causes the axis to shift, causing the gyroscope to rotate in a direction perpendicular to the axis and the applied force. This is because the angular momentum of the gyroscope must remain constant.

3. What are some real-world applications of the gyroscope effect?

The gyroscope effect is used in a variety of applications, including navigation systems in planes, boats, and smartphones, gyroscopic stabilizers in cameras, and gyrocompasses in ships. It is also used in gyroscopic exercise equipment and toys.

4. What factors affect the gyroscope effect?

The gyroscope effect can be affected by the speed of rotation, the mass and shape of the object, the applied force, and the orientation of the object's axis. The gyroscope effect is also influenced by external factors such as friction and air resistance.

5. What are the differences between the gyroscope effect and the gyroscopic stabilization?

While both the gyroscope effect and gyroscopic stabilization involve the rotation of an object, they have different purposes. The gyroscope effect is a physical phenomenon that explains the rotation of an object, while gyroscopic stabilization is a technique that uses the gyroscope effect to stabilize an object's orientation. Gyroscopic stabilization is used in devices such as cameras, drones, and spacecraft to maintain a stable position.

Similar threads

Replies
6
Views
1K
  • Classical Physics
Replies
4
Views
1K
  • Classical Physics
Replies
10
Views
1K
Replies
8
Views
1K
Replies
23
Views
995
Replies
3
Views
929
  • Classical Physics
Replies
1
Views
1K
Replies
14
Views
5K
Replies
41
Views
11K
Replies
11
Views
1K
Back
Top